A bag contains 4 white, 7 black and 5 red balls. 4 balls are drawn with replacement. Then Find the following...












2












$begingroup$



Question: A bag contains 4 white, 7 black and 5 red balls. 4 balls are drawn with replacement. Show that the probability of at least two balls are white is $frac{67}{256}$.*




I have tried in the following way:



Number of white balls = 4

Number of black balls = 7

Number of red balls = 5

Total balls = 16



$P (text{at least $2$ balls drawn are white})$
$= P (2text{ balls drawn are white}) + P (3 text{ balls drawn are white}) +P (4 text{ balls drawn are white}) $
$=frac{^4C_2times ^{12}C_2}{^{16}C_4}+frac{^4C_3times ^{12}C_1}{^{16}C_4}+frac{^4C_4times ^{12}C_0}{^{16}C_4}$



I don't know whether my approach is correct or not.










share|cite|improve this question











$endgroup$












  • $begingroup$
    It looks like you may not have accounted for the fact that, using your first term as an example, you also require the other two balls to be non-white. What does that calculation give you?
    $endgroup$
    – The Count
    Jan 18 '17 at 3:08


















2












$begingroup$



Question: A bag contains 4 white, 7 black and 5 red balls. 4 balls are drawn with replacement. Show that the probability of at least two balls are white is $frac{67}{256}$.*




I have tried in the following way:



Number of white balls = 4

Number of black balls = 7

Number of red balls = 5

Total balls = 16



$P (text{at least $2$ balls drawn are white})$
$= P (2text{ balls drawn are white}) + P (3 text{ balls drawn are white}) +P (4 text{ balls drawn are white}) $
$=frac{^4C_2times ^{12}C_2}{^{16}C_4}+frac{^4C_3times ^{12}C_1}{^{16}C_4}+frac{^4C_4times ^{12}C_0}{^{16}C_4}$



I don't know whether my approach is correct or not.










share|cite|improve this question











$endgroup$












  • $begingroup$
    It looks like you may not have accounted for the fact that, using your first term as an example, you also require the other two balls to be non-white. What does that calculation give you?
    $endgroup$
    – The Count
    Jan 18 '17 at 3:08
















2












2








2





$begingroup$



Question: A bag contains 4 white, 7 black and 5 red balls. 4 balls are drawn with replacement. Show that the probability of at least two balls are white is $frac{67}{256}$.*




I have tried in the following way:



Number of white balls = 4

Number of black balls = 7

Number of red balls = 5

Total balls = 16



$P (text{at least $2$ balls drawn are white})$
$= P (2text{ balls drawn are white}) + P (3 text{ balls drawn are white}) +P (4 text{ balls drawn are white}) $
$=frac{^4C_2times ^{12}C_2}{^{16}C_4}+frac{^4C_3times ^{12}C_1}{^{16}C_4}+frac{^4C_4times ^{12}C_0}{^{16}C_4}$



I don't know whether my approach is correct or not.










share|cite|improve this question











$endgroup$





Question: A bag contains 4 white, 7 black and 5 red balls. 4 balls are drawn with replacement. Show that the probability of at least two balls are white is $frac{67}{256}$.*




I have tried in the following way:



Number of white balls = 4

Number of black balls = 7

Number of red balls = 5

Total balls = 16



$P (text{at least $2$ balls drawn are white})$
$= P (2text{ balls drawn are white}) + P (3 text{ balls drawn are white}) +P (4 text{ balls drawn are white}) $
$=frac{^4C_2times ^{12}C_2}{^{16}C_4}+frac{^4C_3times ^{12}C_1}{^{16}C_4}+frac{^4C_4times ^{12}C_0}{^{16}C_4}$



I don't know whether my approach is correct or not.







probability combinatorics






share|cite|improve this question















share|cite|improve this question













share|cite|improve this question




share|cite|improve this question








edited Jan 18 '17 at 3:11









suomynonA

5,65122557




5,65122557










asked Jan 18 '17 at 2:59









PrimoPrimo

1412




1412












  • $begingroup$
    It looks like you may not have accounted for the fact that, using your first term as an example, you also require the other two balls to be non-white. What does that calculation give you?
    $endgroup$
    – The Count
    Jan 18 '17 at 3:08




















  • $begingroup$
    It looks like you may not have accounted for the fact that, using your first term as an example, you also require the other two balls to be non-white. What does that calculation give you?
    $endgroup$
    – The Count
    Jan 18 '17 at 3:08


















$begingroup$
It looks like you may not have accounted for the fact that, using your first term as an example, you also require the other two balls to be non-white. What does that calculation give you?
$endgroup$
– The Count
Jan 18 '17 at 3:08






$begingroup$
It looks like you may not have accounted for the fact that, using your first term as an example, you also require the other two balls to be non-white. What does that calculation give you?
$endgroup$
– The Count
Jan 18 '17 at 3:08












4 Answers
4






active

oldest

votes


















1












$begingroup$

Note you're sampling without replacement, so each ball drawn can be treated as an independent trial with $(4/16)$ chance of drawing a white ball.



If two are white, any two of the four can be white, so there are ${4choose 2 } = 6$ possibilities, each of which have probability $(4/16)^2(12/16)^2$ so the probability that two are white is $6*(4/16)^2*(12/16)^2.$



Similarly if three are white, there are $4$ ways and the probability is $$4*(4/16)^3*(12/16)^1.$$



And the probability all four are white is $(4/16)^4.$



The total probability that two or more are white is the sum $$frac{6*4^2*12^2 + 4*4^3*12 + 4^4}{16^4} = frac{17152}{16^4} = frac{67}{256} $$



Your approach of using $12 choose 2$, etc is better suited to sampling without replacement.






share|cite|improve this answer









$endgroup$





















    1












    $begingroup$

    Let X be number of white balls during 4 draws, we can see X follows Binomial distribution because of drawing with replacement: Bin(4, 1/4).



    So $P(X=k) = {4 choose k}frac{1}{4}^kfrac{3}{4}^{4-k} $



    $ P(Xge2) = 1 - P(X=0) - P(X=1) $



    Plug in 0 and 1 to the PMF and get the answer $frac{67}{256}$.






    share|cite|improve this answer









    $endgroup$





















      0












      $begingroup$

      Well, the probability that any one of them will be white is $1/8$, correct?
      So if you want at least two of them to be white, then you can find the probability that only one or zero of them will be white, then take the inverse of that.



      $$begin{align*}P(text{one ball will be white}) &= frac18cdotleft(frac78right)^3cdot4=frac12cdotleft(frac78right)^3 \
      P(text{no balls will be white}) &= left(frac78right)^4\
      P(text{at least two will be white})&=1-P(text{one will be white}) - P(text{no balls will be white})end{align*}$$



      Calculate the rest from there.






      share|cite|improve this answer









      $endgroup$





















        0












        $begingroup$

        To follow a more systematic approach that you can use in general, start with
        $$
        p_{;1} = {4 over {16}}quad p_{;2} = {7 over {16}}quad p_{;3} = {5 over {16}}
        $$



        Because draws are with replacement, they are constant at each draw.



        Now, consider that
        $$
        1 = p_{;1} + p_{;2} + p_{;3} = left( {p_{;1} + p_{;2} + p_{;3} } right)^{,4} = left( {p_{;1} + left( {1 - p_{;1} } right)} right)^{,4}
        $$



        From this, applying the binomial (or multinomial) expansion you can compute all
        the concerned probabilities.



        In your case
        $$
        Pleft( {2 le w} right) = sumlimits_{2, le ,k, le 4} {left( matrix{
        4 cr
        k cr} right)p_{;1} ^{,k} left( {1 - p_{;1} } right)^{,4 - k} } = {{67} over {256}}
        $$






        share|cite|improve this answer









        $endgroup$













          Your Answer





          StackExchange.ifUsing("editor", function () {
          return StackExchange.using("mathjaxEditing", function () {
          StackExchange.MarkdownEditor.creationCallbacks.add(function (editor, postfix) {
          StackExchange.mathjaxEditing.prepareWmdForMathJax(editor, postfix, [["$", "$"], ["\\(","\\)"]]);
          });
          });
          }, "mathjax-editing");

          StackExchange.ready(function() {
          var channelOptions = {
          tags: "".split(" "),
          id: "69"
          };
          initTagRenderer("".split(" "), "".split(" "), channelOptions);

          StackExchange.using("externalEditor", function() {
          // Have to fire editor after snippets, if snippets enabled
          if (StackExchange.settings.snippets.snippetsEnabled) {
          StackExchange.using("snippets", function() {
          createEditor();
          });
          }
          else {
          createEditor();
          }
          });

          function createEditor() {
          StackExchange.prepareEditor({
          heartbeatType: 'answer',
          autoActivateHeartbeat: false,
          convertImagesToLinks: true,
          noModals: true,
          showLowRepImageUploadWarning: true,
          reputationToPostImages: 10,
          bindNavPrevention: true,
          postfix: "",
          imageUploader: {
          brandingHtml: "Powered by u003ca class="icon-imgur-white" href="https://imgur.com/"u003eu003c/au003e",
          contentPolicyHtml: "User contributions licensed under u003ca href="https://creativecommons.org/licenses/by-sa/3.0/"u003ecc by-sa 3.0 with attribution requiredu003c/au003e u003ca href="https://stackoverflow.com/legal/content-policy"u003e(content policy)u003c/au003e",
          allowUrls: true
          },
          noCode: true, onDemand: true,
          discardSelector: ".discard-answer"
          ,immediatelyShowMarkdownHelp:true
          });


          }
          });














          draft saved

          draft discarded


















          StackExchange.ready(
          function () {
          StackExchange.openid.initPostLogin('.new-post-login', 'https%3a%2f%2fmath.stackexchange.com%2fquestions%2f2102476%2fa-bag-contains-4-white-7-black-and-5-red-balls-4-balls-are-drawn-with-replacem%23new-answer', 'question_page');
          }
          );

          Post as a guest















          Required, but never shown

























          4 Answers
          4






          active

          oldest

          votes








          4 Answers
          4






          active

          oldest

          votes









          active

          oldest

          votes






          active

          oldest

          votes









          1












          $begingroup$

          Note you're sampling without replacement, so each ball drawn can be treated as an independent trial with $(4/16)$ chance of drawing a white ball.



          If two are white, any two of the four can be white, so there are ${4choose 2 } = 6$ possibilities, each of which have probability $(4/16)^2(12/16)^2$ so the probability that two are white is $6*(4/16)^2*(12/16)^2.$



          Similarly if three are white, there are $4$ ways and the probability is $$4*(4/16)^3*(12/16)^1.$$



          And the probability all four are white is $(4/16)^4.$



          The total probability that two or more are white is the sum $$frac{6*4^2*12^2 + 4*4^3*12 + 4^4}{16^4} = frac{17152}{16^4} = frac{67}{256} $$



          Your approach of using $12 choose 2$, etc is better suited to sampling without replacement.






          share|cite|improve this answer









          $endgroup$


















            1












            $begingroup$

            Note you're sampling without replacement, so each ball drawn can be treated as an independent trial with $(4/16)$ chance of drawing a white ball.



            If two are white, any two of the four can be white, so there are ${4choose 2 } = 6$ possibilities, each of which have probability $(4/16)^2(12/16)^2$ so the probability that two are white is $6*(4/16)^2*(12/16)^2.$



            Similarly if three are white, there are $4$ ways and the probability is $$4*(4/16)^3*(12/16)^1.$$



            And the probability all four are white is $(4/16)^4.$



            The total probability that two or more are white is the sum $$frac{6*4^2*12^2 + 4*4^3*12 + 4^4}{16^4} = frac{17152}{16^4} = frac{67}{256} $$



            Your approach of using $12 choose 2$, etc is better suited to sampling without replacement.






            share|cite|improve this answer









            $endgroup$
















              1












              1








              1





              $begingroup$

              Note you're sampling without replacement, so each ball drawn can be treated as an independent trial with $(4/16)$ chance of drawing a white ball.



              If two are white, any two of the four can be white, so there are ${4choose 2 } = 6$ possibilities, each of which have probability $(4/16)^2(12/16)^2$ so the probability that two are white is $6*(4/16)^2*(12/16)^2.$



              Similarly if three are white, there are $4$ ways and the probability is $$4*(4/16)^3*(12/16)^1.$$



              And the probability all four are white is $(4/16)^4.$



              The total probability that two or more are white is the sum $$frac{6*4^2*12^2 + 4*4^3*12 + 4^4}{16^4} = frac{17152}{16^4} = frac{67}{256} $$



              Your approach of using $12 choose 2$, etc is better suited to sampling without replacement.






              share|cite|improve this answer









              $endgroup$



              Note you're sampling without replacement, so each ball drawn can be treated as an independent trial with $(4/16)$ chance of drawing a white ball.



              If two are white, any two of the four can be white, so there are ${4choose 2 } = 6$ possibilities, each of which have probability $(4/16)^2(12/16)^2$ so the probability that two are white is $6*(4/16)^2*(12/16)^2.$



              Similarly if three are white, there are $4$ ways and the probability is $$4*(4/16)^3*(12/16)^1.$$



              And the probability all four are white is $(4/16)^4.$



              The total probability that two or more are white is the sum $$frac{6*4^2*12^2 + 4*4^3*12 + 4^4}{16^4} = frac{17152}{16^4} = frac{67}{256} $$



              Your approach of using $12 choose 2$, etc is better suited to sampling without replacement.







              share|cite|improve this answer












              share|cite|improve this answer



              share|cite|improve this answer










              answered Jan 18 '17 at 3:09









              spaceisdarkgreenspaceisdarkgreen

              33.4k21753




              33.4k21753























                  1












                  $begingroup$

                  Let X be number of white balls during 4 draws, we can see X follows Binomial distribution because of drawing with replacement: Bin(4, 1/4).



                  So $P(X=k) = {4 choose k}frac{1}{4}^kfrac{3}{4}^{4-k} $



                  $ P(Xge2) = 1 - P(X=0) - P(X=1) $



                  Plug in 0 and 1 to the PMF and get the answer $frac{67}{256}$.






                  share|cite|improve this answer









                  $endgroup$


















                    1












                    $begingroup$

                    Let X be number of white balls during 4 draws, we can see X follows Binomial distribution because of drawing with replacement: Bin(4, 1/4).



                    So $P(X=k) = {4 choose k}frac{1}{4}^kfrac{3}{4}^{4-k} $



                    $ P(Xge2) = 1 - P(X=0) - P(X=1) $



                    Plug in 0 and 1 to the PMF and get the answer $frac{67}{256}$.






                    share|cite|improve this answer









                    $endgroup$
















                      1












                      1








                      1





                      $begingroup$

                      Let X be number of white balls during 4 draws, we can see X follows Binomial distribution because of drawing with replacement: Bin(4, 1/4).



                      So $P(X=k) = {4 choose k}frac{1}{4}^kfrac{3}{4}^{4-k} $



                      $ P(Xge2) = 1 - P(X=0) - P(X=1) $



                      Plug in 0 and 1 to the PMF and get the answer $frac{67}{256}$.






                      share|cite|improve this answer









                      $endgroup$



                      Let X be number of white balls during 4 draws, we can see X follows Binomial distribution because of drawing with replacement: Bin(4, 1/4).



                      So $P(X=k) = {4 choose k}frac{1}{4}^kfrac{3}{4}^{4-k} $



                      $ P(Xge2) = 1 - P(X=0) - P(X=1) $



                      Plug in 0 and 1 to the PMF and get the answer $frac{67}{256}$.







                      share|cite|improve this answer












                      share|cite|improve this answer



                      share|cite|improve this answer










                      answered Jan 18 '17 at 3:40









                      Ryan Y.Ryan Y.

                      413




                      413























                          0












                          $begingroup$

                          Well, the probability that any one of them will be white is $1/8$, correct?
                          So if you want at least two of them to be white, then you can find the probability that only one or zero of them will be white, then take the inverse of that.



                          $$begin{align*}P(text{one ball will be white}) &= frac18cdotleft(frac78right)^3cdot4=frac12cdotleft(frac78right)^3 \
                          P(text{no balls will be white}) &= left(frac78right)^4\
                          P(text{at least two will be white})&=1-P(text{one will be white}) - P(text{no balls will be white})end{align*}$$



                          Calculate the rest from there.






                          share|cite|improve this answer









                          $endgroup$


















                            0












                            $begingroup$

                            Well, the probability that any one of them will be white is $1/8$, correct?
                            So if you want at least two of them to be white, then you can find the probability that only one or zero of them will be white, then take the inverse of that.



                            $$begin{align*}P(text{one ball will be white}) &= frac18cdotleft(frac78right)^3cdot4=frac12cdotleft(frac78right)^3 \
                            P(text{no balls will be white}) &= left(frac78right)^4\
                            P(text{at least two will be white})&=1-P(text{one will be white}) - P(text{no balls will be white})end{align*}$$



                            Calculate the rest from there.






                            share|cite|improve this answer









                            $endgroup$
















                              0












                              0








                              0





                              $begingroup$

                              Well, the probability that any one of them will be white is $1/8$, correct?
                              So if you want at least two of them to be white, then you can find the probability that only one or zero of them will be white, then take the inverse of that.



                              $$begin{align*}P(text{one ball will be white}) &= frac18cdotleft(frac78right)^3cdot4=frac12cdotleft(frac78right)^3 \
                              P(text{no balls will be white}) &= left(frac78right)^4\
                              P(text{at least two will be white})&=1-P(text{one will be white}) - P(text{no balls will be white})end{align*}$$



                              Calculate the rest from there.






                              share|cite|improve this answer









                              $endgroup$



                              Well, the probability that any one of them will be white is $1/8$, correct?
                              So if you want at least two of them to be white, then you can find the probability that only one or zero of them will be white, then take the inverse of that.



                              $$begin{align*}P(text{one ball will be white}) &= frac18cdotleft(frac78right)^3cdot4=frac12cdotleft(frac78right)^3 \
                              P(text{no balls will be white}) &= left(frac78right)^4\
                              P(text{at least two will be white})&=1-P(text{one will be white}) - P(text{no balls will be white})end{align*}$$



                              Calculate the rest from there.







                              share|cite|improve this answer












                              share|cite|improve this answer



                              share|cite|improve this answer










                              answered Jan 18 '17 at 3:10









                              Drew ChristensenDrew Christensen

                              404211




                              404211























                                  0












                                  $begingroup$

                                  To follow a more systematic approach that you can use in general, start with
                                  $$
                                  p_{;1} = {4 over {16}}quad p_{;2} = {7 over {16}}quad p_{;3} = {5 over {16}}
                                  $$



                                  Because draws are with replacement, they are constant at each draw.



                                  Now, consider that
                                  $$
                                  1 = p_{;1} + p_{;2} + p_{;3} = left( {p_{;1} + p_{;2} + p_{;3} } right)^{,4} = left( {p_{;1} + left( {1 - p_{;1} } right)} right)^{,4}
                                  $$



                                  From this, applying the binomial (or multinomial) expansion you can compute all
                                  the concerned probabilities.



                                  In your case
                                  $$
                                  Pleft( {2 le w} right) = sumlimits_{2, le ,k, le 4} {left( matrix{
                                  4 cr
                                  k cr} right)p_{;1} ^{,k} left( {1 - p_{;1} } right)^{,4 - k} } = {{67} over {256}}
                                  $$






                                  share|cite|improve this answer









                                  $endgroup$


















                                    0












                                    $begingroup$

                                    To follow a more systematic approach that you can use in general, start with
                                    $$
                                    p_{;1} = {4 over {16}}quad p_{;2} = {7 over {16}}quad p_{;3} = {5 over {16}}
                                    $$



                                    Because draws are with replacement, they are constant at each draw.



                                    Now, consider that
                                    $$
                                    1 = p_{;1} + p_{;2} + p_{;3} = left( {p_{;1} + p_{;2} + p_{;3} } right)^{,4} = left( {p_{;1} + left( {1 - p_{;1} } right)} right)^{,4}
                                    $$



                                    From this, applying the binomial (or multinomial) expansion you can compute all
                                    the concerned probabilities.



                                    In your case
                                    $$
                                    Pleft( {2 le w} right) = sumlimits_{2, le ,k, le 4} {left( matrix{
                                    4 cr
                                    k cr} right)p_{;1} ^{,k} left( {1 - p_{;1} } right)^{,4 - k} } = {{67} over {256}}
                                    $$






                                    share|cite|improve this answer









                                    $endgroup$
















                                      0












                                      0








                                      0





                                      $begingroup$

                                      To follow a more systematic approach that you can use in general, start with
                                      $$
                                      p_{;1} = {4 over {16}}quad p_{;2} = {7 over {16}}quad p_{;3} = {5 over {16}}
                                      $$



                                      Because draws are with replacement, they are constant at each draw.



                                      Now, consider that
                                      $$
                                      1 = p_{;1} + p_{;2} + p_{;3} = left( {p_{;1} + p_{;2} + p_{;3} } right)^{,4} = left( {p_{;1} + left( {1 - p_{;1} } right)} right)^{,4}
                                      $$



                                      From this, applying the binomial (or multinomial) expansion you can compute all
                                      the concerned probabilities.



                                      In your case
                                      $$
                                      Pleft( {2 le w} right) = sumlimits_{2, le ,k, le 4} {left( matrix{
                                      4 cr
                                      k cr} right)p_{;1} ^{,k} left( {1 - p_{;1} } right)^{,4 - k} } = {{67} over {256}}
                                      $$






                                      share|cite|improve this answer









                                      $endgroup$



                                      To follow a more systematic approach that you can use in general, start with
                                      $$
                                      p_{;1} = {4 over {16}}quad p_{;2} = {7 over {16}}quad p_{;3} = {5 over {16}}
                                      $$



                                      Because draws are with replacement, they are constant at each draw.



                                      Now, consider that
                                      $$
                                      1 = p_{;1} + p_{;2} + p_{;3} = left( {p_{;1} + p_{;2} + p_{;3} } right)^{,4} = left( {p_{;1} + left( {1 - p_{;1} } right)} right)^{,4}
                                      $$



                                      From this, applying the binomial (or multinomial) expansion you can compute all
                                      the concerned probabilities.



                                      In your case
                                      $$
                                      Pleft( {2 le w} right) = sumlimits_{2, le ,k, le 4} {left( matrix{
                                      4 cr
                                      k cr} right)p_{;1} ^{,k} left( {1 - p_{;1} } right)^{,4 - k} } = {{67} over {256}}
                                      $$







                                      share|cite|improve this answer












                                      share|cite|improve this answer



                                      share|cite|improve this answer










                                      answered Jan 22 at 19:03









                                      G CabG Cab

                                      19.9k31340




                                      19.9k31340






























                                          draft saved

                                          draft discarded




















































                                          Thanks for contributing an answer to Mathematics Stack Exchange!


                                          • Please be sure to answer the question. Provide details and share your research!

                                          But avoid



                                          • Asking for help, clarification, or responding to other answers.

                                          • Making statements based on opinion; back them up with references or personal experience.


                                          Use MathJax to format equations. MathJax reference.


                                          To learn more, see our tips on writing great answers.




                                          draft saved


                                          draft discarded














                                          StackExchange.ready(
                                          function () {
                                          StackExchange.openid.initPostLogin('.new-post-login', 'https%3a%2f%2fmath.stackexchange.com%2fquestions%2f2102476%2fa-bag-contains-4-white-7-black-and-5-red-balls-4-balls-are-drawn-with-replacem%23new-answer', 'question_page');
                                          }
                                          );

                                          Post as a guest















                                          Required, but never shown





















































                                          Required, but never shown














                                          Required, but never shown












                                          Required, but never shown







                                          Required, but never shown

































                                          Required, but never shown














                                          Required, but never shown












                                          Required, but never shown







                                          Required, but never shown







                                          Popular posts from this blog

                                          Can a sorcerer learn a 5th-level spell early by creating spell slots using the Font of Magic feature?

                                          Does disintegrating a polymorphed enemy still kill it after the 2018 errata?

                                          A Topological Invariant for $pi_3(U(n))$